summaryrefslogtreecommitdiff
diff options
context:
space:
mode:
-rw-r--r--exercise06/exercise06.tex126
1 files changed, 124 insertions, 2 deletions
diff --git a/exercise06/exercise06.tex b/exercise06/exercise06.tex
index 954357e..9054ced 100644
--- a/exercise06/exercise06.tex
+++ b/exercise06/exercise06.tex
@@ -14,10 +14,38 @@
%%%%%%%%%%%%%%%%%%%%%%%%%%%%%%%%%%%%%%%%%%%%%%%%%%%%%%%%%%%%%%%%%%%%%%%%%%%%%%%
-\begin{question}[subtitle={Decibel}]
- Proof mathematically that all poles of the FIR filter are $0$!
+\begin{question}[subtitle={IIR Filter}]
+ The following IIR filter is given.
+ \begin{figure}[H]
+ \centering
+ \begin{circuitikz}
+ \draw[o-] (-1,0) node[left, align=right]{$\underline{x}[n]$} -- (0,0);
+ \draw (0,-3) node[adder](Add1){};
+ \draw (2,-3) node[adder](Add2){};
+ \draw (0,0) to[amp,l=$\underline{b}_0$,>,-] (Add1.north) node[inputarrow,rotate=-90]{};
+ \draw (0,0) to[short,*-] (2,0) to[twoport,t=$z^{-1}$,>,-] (Add2.north) node[inputarrow,rotate=-90]{};
+ \draw (Add1.east) to[short] (Add2.west) node[inputarrow,rotate=0]{};
+ \draw[-latex] (Add2.east) to[short] (4,-3) node[right, align=left]{$\underline{y}[n]$};
+ \draw (3,-3) to[short,*-] (3,-6) to[twoport,t=$z^{-1}$,>,-] (0,-6) to[amp,l=$\underline{a}_0$,>,-] (Add1.south) node[inputarrow,rotate=90]{};
+ \end{circuitikz}
+ \end{figure}
+ with:
+ \begin{itemize}
+ \item $\underline{a}_0 = 0.5$
+ \item $\underline{b}_0 = 2$
+ \end{itemize}
\begin{tasks}
+ \task
+ Give the block diagram of the filter!
+ \task
+ Give the differential equation of the filter!
+ \task
+ How much is the filter order?
+ \task
+ Is the filter stable?
+ \task
+ Plot the amplitude and phase response between $0$ and $\pi$.
\end{tasks}
\end{question}
@@ -28,6 +56,100 @@
%%%%%%%%%%%%%%%%%%%%%%%%%%%%%%%%%%%%%%%%%%%%%%%%%%%%%%%%%%%%%%%%%%%%%%%%%%%%%%%
+\begin{question}[subtitle={FIR Filter}]
+ An FIR filter with following coefficients is given.
+ \begin{itemize}
+ \item $b_0 = 1$.
+ \item $b_1 = 0.5 + j \cdot 1$.
+ \item $b_2 = 2$.
+ \end{itemize}
+
+ The sampling rate of the digital system is \SI{2}{MHz}.
+
+ \begin{tasks}
+ \task
+ Give the block diagram of the filter!
+ \task
+ Give the transfer function of the filter!
+ \task
+ Give the differential equation of the filter!
+ \task
+ How much is the filter order?
+ \task
+ Plot the amplitude and phase response between \SI{-1}{MHz} and \SI{1}{MHz}.
+ \task
+ Proof mathematically that all poles of the FIR filter are $0$!
+ \end{tasks}
+\end{question}
+
+\begin{solution}
+ \begin{tasks}
+ \end{tasks}
+\end{solution}
+
+
+%%%%%%%%%%%%%%%%%%%%%%%%%%%%%%%%%%%%%%%%%%%%%%%%%%%%%%%%%%%%%%%%%%%%%%%%%%%%%%%
+\begin{question}[subtitle={Down-sampling}]
+ An analogue signal $x(t)$ is digitized (sampled and quantized).
+ \begin{equation*}
+ x(t) = \sin\left(2 \pi \cdot \SI{96}{kHz} \cdot t\right)
+ \end{equation*}
+ The signal has been sampled by a \SI{8}{bit}-ADC at \SI{7.68}{MHz}.
+
+ The signal $x[n]$ is decimated by $N = 40$.
+
+ \begin{tasks}
+ \task
+ How much is the sampling rate of the decimated signal?
+ \task
+ Is the signal suitable to be decimated by $N = 40$? Explain why! What is the criterion?
+ \task
+ What is the optimal sampling phase?
+ \task
+ Explain the effect on the spectrum caused by down-sampling!
+ \task
+ How much is the processing gain? How much is the effective number of bits?
+ \end{tasks}
+\end{question}
+
+\begin{solution}
+ \begin{tasks}
+ \end{tasks}
+\end{solution}
+
+
+%%%%%%%%%%%%%%%%%%%%%%%%%%%%%%%%%%%%%%%%%%%%%%%%%%%%%%%%%%%%%%%%%%%%%%%%%%%%%%%
+\begin{question}[subtitle={FFT}]
+ A series of the samples in the time-domain is given:
+ \begin{equation*}
+ x[n] = \left[2 \underline{-0.5} 1 -2 \right]
+ \end{equation*}
+
+ \begin{remark}
+ The underline marks the sample at $n = 0$.
+ \end{remark}
+
+ \begin{tasks}
+ \task
+ Calculate the DFT for $k = 0, \ldots, 3$!
+ \task
+ Calculate the FFT using the Cooley-Tuckey FFT algorithm!
+ \task
+ Compare the number of multiply-accumulate operations necessary for both methods in a) and b)!
+ \task
+ Draw the butterfly graph!
+ \task
+ Give th primitive roots of unity for each sub-FFT in the butterfly graph!
+ \end{tasks}
+\end{question}
+
+\begin{solution}
+ %The signal is periodic with $N = 4$.
+ \begin{tasks}
+ \end{tasks}
+\end{solution}
+
+%%%%%%%%%%%%%%%%%%%%%%%%%%%%%%%%%%%%%%%%%%%%%%%%%%%%%%%%%%%%%%%%%%%%%%%%%%%%%%%
%\begin{question}[subtitle={Decibel}]
% \begin{tasks}
% \end{tasks}